Anda di halaman 1dari 45

Place Values of Alphabets - Reasoning Shortcuts F sponsored links riends, today we are going to learn about the basic

rule for Analytical Reasoning. The Place Values of English Alphabets. These will help you in answering the questions related to Coding and Decoding and Analogy... Yea, I know all of you know alphabets and me too don't want to teach them to you now. But what about the place values? Of course, even a 5th grader will tell the answer by counting if you ask him What is the place value of S ?". But the problem is, you should react in fractions of a second. Because you cant spend more than 30 seconds for these type of questions. That means, you should learn by heart. Below we've given you the alphabets with place values, just read them for 10 to 15 times so that the values of these alphabets will register permanently in your mind. 1234567 ABCDEFG 8 9 10 11 12 13 14 HIJKLMN 15 16 17 18 19 20 21 OPQRSTU 22 23 24 25 26 VWXYZ How to remember them in a Simple Way? All of you easily can say the place values of first 4 (a, b, c, d) and last 3 (x, y, z) alphabets. Remember F6 key in your Keyboard. Thats it, the place value of F is 6.Have you ever Noticed the 7 in G? Check the below figure once, thats it.. G's value is 7. so as the numbers eigHt and nIne In this way, you can make your won shortcut rules to remember the place values. Note : My Stupid Brother (one of the moderators of this blog) made a unique technique to recall the place values. His code word is E mail Jothy. In short E JOTY......... These represents the place values which are multiples of 5. E-5 J - 10 O - 15 T - 20 Y - 25 With the help of this list, he can recall place values of all the 26 alphabets in Seconds. Thats all for now friends. Hope you enjoyed this post. From our next post we shall discuss the shortcut techniques of Coding and Decoding using Place Values. You can get more Reasoning Shortcuts here. Classification / Odd Man Out Series Problems with Explnations Alphabets friends, Classification / Odd Man out Series is one of the important topic in Reasoning Section. In these type of questions they just give you 5 options and you should find the odd man out from the given set. I mean, if you are given 5 options like Parrot, Crow, vulture, Sparrow and Penguin and ask you to find the Odd man out, then your answer should be Penguin (because all the remaining birds can fly)... Ofcourse, some of you may say "Why not Vulture is the answer? Because its the only bird which eats dead bodies (actually one of my friends asked this question while writing this article). Point.... "May be other birds too will eat dead insects if they starved" I answered. "Then penguins too can fly for short distances" she replied. Point Again!!! So what will be the correct answer? After all you are not going to argue with the examiner who evaluates your answer sheet (or Optical Marks Reader ofcourse). So, here you should keep in mind that OVER THINKING should be avoided while answering these type of questions. Think twice before marking your answer. Now lets go to the details. To made the concepts clear, we've divided this topic into 3 sections. Those are 1. Alphabets 2. Numbers 3. Words. Today, we shall discuss about Alphabets... Before reading this post, it is advisable to read about the place values of Alphabets here. Single Alphabets : If single alphabets are given, then you should follow the order of checking 1. Vowels first. If you are unable to figure odd ones, then check for 2. mirror images, later 3. water images and finally check for the 4. place values so that you may get any clues with the number assiciated with that specific alphabet. Example Problems : 1. A 2. B 3. C 4. D 5. G Solution : Obviously here the answer is A. Because its the only Vowel in the list. 1. A 2. C 3. E 4. H 5. J Solution : Here the answer is H. Because, the Difference between alphabets is 2. 1. D 2. E 3. F 4. G

5. H Solution : Here the Odd one is H, because it is the only Mirror Image in the List. (There are 2 water images D and H) Alphabet Groups If multiple Alphabets are given, you should follow the order of checking, 1. Alphabetical order of given alphabets 2. Distance / Gap between the given alphabets 3. Vowel / Consonant differences Example Problems : 1. ABC 2. BCD 3. CDE 4. DEF 5. EFG Solution : : Check the Order : ABC... BCD... CDE... DEF... EFG... Well, here all alphabets are in an order.. So we cant find out the Odd one with this logic. Check the Gap : All alphabets carrying equal number differences / Gaps.. So this technique too of no use. Check for the Vowels. In the group 1, One vowel is there (A) In the group 2, No vowel In the group 3, One vowel is there (E) In the group 4, One vowel is there (E) In the group 5, One vowel is there (E) So, answer should be the Group with NO VOWELS.. Group 2 1. AE 2. GM 3. MQ 4. VZ 5. LP Here you should note that AE are the only vowels in the given group. So maximum number of people choose Option 1 as their answers. BUT, according to our order given above, we should first check for the alphabetical order... no clue..... later GAP. Check the difference fro Ato E and G to M.... AbcdE GhijklM MnopQ VwxyZ LmnoP Here, except the second pair all the remaining pairs are having 3 letters in between the given alphabets. But the second pair consists of 5 alphabets between G and M. So this is the Odd one. So the answer is Option 2 (not Option 1). 1. DF 2. TV 3. VX 4. NP 5. HJ Solution : 1. DeF 2. TuV 3. VwX 4. NoP 5. HiJ Here all the given pairs are having ONE LETTER between them. So now we shall check those alphabets for clue. e, u, w, o, i.. .Here except w all the remaining alphabets are vowels.So, obviously the odd one is the 3rd pair. 1. AFH 2. AHL 3. AJM 4. ARW 5. ANR Well this is little complected one... have a close look... We can't find anything with Alphabetical order.. So try GAPs... A____F_H A______H___L A________J__M A________________R____W A____________N___R So, A4F1H A6H3L A8J2M A 16 R 4 W A 12 N 3 R No Clue :(

Try adding the numbers we got in gaps..... 4+1 = 5 6+3 = 9 8+2 = 10 16+4 = 20 12+3 = 15 No Clue :( Subtract the numbers, 4-1 = 3 6-3 = 3 8-2 = 6 16-4 = 12 12-3 = 9 No Clue :( Multiply the numbers 4*1 = 4 6*3 = 18 8*2 = 16 16*4 = 6 12*3 = 36 ................ hey see, are you able to figure out anything? All numbers except 18 are square numbers. So option 2 is the Odd one.And remember, for the purpose of explanation, we gave detailed step by step procedure here. For in your exam hall, you should figure out the option / procedure in a single glance. With good practice only you can achieve that state. 1. ACF 2. ADI 3. AGG 4. AFW 5. AEO Solution : A1C2F A2D4I A 5 G 25 G (Note : After Z you should again start with A and continue your counting) A 4 F 16 W A3E9O With a glance, we can find out that 2's square is 4 5's square is 25 4's square is 16 and 3 's square is 9 So, here the odd one is the Option 1. 1. VT 2. MQ 3. PR 4. DF 5. FG Solution : Here except Option 1 (VT) all remaining alphabets are in alphabetical order. VT is in reverse order. So this is the ODD one. 1. LO 2. RU 3. BE 4. FI 5. AD Solution : Here, except the 5th option, all the remaining groups having VOWELs (O, U, E, I) in the second position. But the 5th group contains VOWEL (A) in the first position. So, it is the Odd one. 1. LP 2. SW 3. QU 4. MR 5. AE Solution : Here, except the 4th group, all the remaining groups contains only 3 alphabets between them. But the 4th group has 4 alphabets between M and R. 1. BF 2. NR 3. WS 4. TX 5. LP Solution : 3rd groups is reverse in Alphabetical order. So, it is the odd one. 1. 2. NQ 2. HKP

3. RWZ 4. JMR 5. KNS Solution : 1. 2. 3. 4. 5. N2Q4V H2K4P R4W2Z J2M4R K2N4S

Here obviously the Odd one is option 3 Mathematical Operations friends, today we shall discuss another important model of Reasoning Section Mathematical Operations. Of course, even these questions are asked under reasoning section these are mainly dependent on mathematical operations. All calculations will be performed using BODMAS Rule. We can divide these problems in three types.Those are, 1. Symbol Substitution 2. Mathematical Logic 3. Interchange of Signs & Numbers Now lets discuss these types with detailed examples. 1st Type : Symbol Substitution In this type, to find the value of the given expression you should replace the symbols by mathematical operations. Then apply the BODMAS Rule (i.e., Brackets, of, Division,Multiplication, Addition, Subtraction). For the detailed understanding, lets have a look at an example Example : 42% (9-2) + 6 x 3 - 4 Solution : 42 % (9-2) + 6 x 3 - 4 = 42 % 7 + 6 x 3 - 4 =6+6x3-4 = 6 + 18 - 4 = 24 - 4 = 20 2nd Type : Mathematical Logic In this type, they just give you some logical statements followed by some mathematical operations. You should solve the given mathematical operations with the help of given logical statements. Example : If % stands for greater than, x stands for addition, + stands for division, - stands for equal to, > stands for multiplication, = stands for less than, < stands for minus, then which of the following alternatives is correct ? 1. 3 + 2 < 4 % 6 > 3 x 2 2. 3 x 2 < 4 % 6 + 3 < 2 3. 3 > 2 < 4 - 6 x 3 x 2 4. 3 x 2 x 4 = 6 + 3 < 2 Usually for these type of questions people tend to make direct decisions with the help of given statements. But confusion arises when you tried to coordinate given mathematical symbols with the logical statements mentioned above. It is because lack of coordination between eyes and brain. So, to avoid confusion you just should make a note of given logical statements in simple mathematical equations format. So that it will be easier for your eye to pick the correct operation. by using above logical statements, you can use a rough sketch like this, now you can calculate the values of given operations mentally by picking the right hand side symbols instead of left hand side. Or you just can replace the symbols by taking the reference of above rough sketch. Solution : Using the symbols correctly as given in the above operations, for the option (2) you will get, 3+2-4>6%3-2. Using BODMAS rule in this inequality, we will get 5-4>2-2 or, 1>0, which is correct. 3rd Type : Interchange of Signs & Numbers In this type they just give you some random set of numbers and mathematical symbols. And then ask you to choose the correct of symbols to fit the given equation. Example : Select the correct set of symbols which will fit in the given equation 5 0 3 5 = 20 1. x, x, x 2. =, +, x 3. x, +, x 4. +, -, x Solution : Using the set of symbols given in options, you will get the Option 2 as the correct answer. Using the set of symbols given in option 2, you will get the equation, 5-0+3x5 = 20 Using BODMAS rule you will get,

5-0+15=20 or 20-0 = 20 which is correct Now lets have a look at some examples, 1. '<' means subtraction, '>' means Addition, '+' means Multiplication and '$' means Division. Then what will be the value of 27 > 81 $ 9 < 6 ? Solution : 27+81%9-6 = 27+9-6 = 36-6 = 30 2. If % means +, - means x, x means -, + means %, then what will be the value of 15 - 2 % 900 + 90 x 100 ? Solution : 15x2+900%90-100 = 30 + 10 - 100 = -60 3. Q means Addition, J means Multiplication, T means Subtraction, K means Division then what is the value of 30K2Q3J6T5 ? Solution : 3%2+3x6-5 = 15 + 18 - 5 = 28 4. P means x, R means +, T means %, S means -, then what will be the value of 18T3P9S8R6 ? Solution : By substituting the given symbols, you will get the answer 52 5. P means Multiplication, T means Subtraction, M means Addition, B means Division then what will be the value of 28B7P8T6M4 ? Solution : 28 % 7 x8 - 6 + 4 =4x8-6+4 = 32 - 6 + 4 = 36 - 6 = 30 Thats all for now friends. In our next post we shall discuss some more problems of Mathematical Operations with higher difficulty level. Happy Reading : Verbal Analogy Shortcuts - Introduction friends, generally an analogy is a parallel that is drawn between two different, but adequately similar events, situations of circumstances. In Verbal Analogy questions they will ask you to determine the relationship between a pair of words and then ecognize a similar or parallel relationship in a different pair of words. You are given a pair of words and must choose from the four pairs given as answer choices another pair exhibiting the same relationship. Therefore a test on analogy is a test of one's language / vocabulary and also a test of one's ability to reason out relationships. There are basically a few common types of analogy questions. An effort has been made to gather as many types as possible. Howewer, there is always a possibility of having more types. That is why "Analogy" is a test to assess the psycho-linguistic abilities of the candidates appearing fro a particular competitive examination. Generally, in examinations, the directions for the questions on analogy may appear as follows : Directions : In Each of the following questions, a related pair of words or phrases is followed by lettered pairs of words or phrases. Select the lettered pair that best expresses a relationship similar to that expressed in the original pair. Example : 1. BEE : HIVE :: 1. horse : carriage 2. rider : bicycle 3. sheep : flock 4. cow : barn 2. BEE : APARY :: 1. horse : carriage 2. rider : bicycle 3. sheep : flock 4. cow : barn see here, the answer in both cases is 'D' in question 1, a HIVE is the home for BEES, just as a barn is a home for cows. Similarly, an APIARY (also known as bee yard) is used to house bees, just as barn is used to house cows..Have a look at following test exercise. just to know if you could identify an analogy item.Check how many of the below are possible pairs for analogies (i.e., there exists some relationship between the words) 1. OAR : BOAT 2. KNIFE : TRUCK 3. HORSE : TREE 4. SYRUP : APPLE 5. SCRIBBLE : WRITE 6. UNICORN : CATASTROPHE 7. PASTEL : COLOR 8. RESTIVE : AGREEMENT 9. MELT : LIQUID 10. SYMPHONY : MUSIC Now, analyse your mistakes and know why you went wrong... You might have gone wrong basically for two reasons : 1. Lexical Problem 2. Logical Relationship Problem If you have a Lexical problem (a Lexicon is nothing but a Dictionary), you have to imporve your vocabulary. If your problem is a logical relationship problem, then you must follow a good strategy to answer the 'analogy questions' (no need to be worried.. it will come with a little practice).

How to Solve Verbal Anology Problems of Reasoning ? Note : This post is the Lesson 2 of our Verbal Analogies Shortcuts series. It is advisable to read the Introductory Lesson 1 of Verbal Analogies here before reading this post. riends, in our last post we have discussed the basics of Verbal Analogies and today we shall discuss he strategy you should follow to solve verbal analogy questions. Verbal analogies demand that you, first of all, identify the relationship between tow words (the head pair) and find another pair of words that parallel as closely as possible that same relationship. 1. The first step in analysing original analogous pair of words (the head pair) is to describe the connection between the words of the pair. The best way to do this is to formulate a sentence that express the connection between the words. Since the purpose of this sentence is to analyse or diagnose the nature of the connection between the words. Since the purpose of this sentence is to analyze or diagnose the nature of the connection between the words of the head pair, we may call it Diagnostic Sentence or DXS for short. 2. After formulating the DXS, test each answer choice to find the pair of words that best fit the sentence. Confused? Have a look at an example..... ECSTASY : PLEASURE :: 1. hatred : affection 2. condemnation : approval 3. rage : anger 4. difficult : understanding 5. privacy : invasion Now, the first step is to formulate a DXS. ECSTASY means "great or overwhelming PLEASURE," so the DXS might be "ECSTASY is treme PLEASURE" The second step is to test each of the answer choices with the DXS.....................is extreme.................... 1. Hatred is extreme affection 2. Condemnation is extreme approval 3. Rage is extreme anger 4. Difficulty is extreme understanding 5. Privacy is extreme invasion Here, option 3 is the best answer. Because for only Option 3, the pair of words correctly fit the DXS. Now lets have a look at another example. ACCIDENTAL : INTENTION :: 1. Voluntary : Requirement 2. Anticipated : Performance 3. Interesting : Feeling 4. Practical : Knowledge 5. Insane : Connection DXS : Somethign that is ACCIDENTAL is done without INTENTION.Now, test each of the answer choices with the diagnostic sentence. Something that is ............ is done without ......................... 1. Something that is voluntary is done without requirement 2. Something that is anticipated is done withoutperformance 3. Something that is interesting is done without feeling 4. Something that is practical is done without knowledge 5. Something that is insane is done without connection here option 1 is the answer, and it is the only logical substitution that makes scene in this DXS.Thats all for now friends. In our next lesson we shall discuss some common AnalogousRelationships. All the best and Happy Reading :) Analogy (Advanced) - Reasoning Shortcuts A Analogy means to compare the similarities / dissimilarities among the given things. In this type of questions, they will give you a sentence and options. According to the sentence you should find out the correct option. Have a look at some examples to understand Analogy more clearly... "Bank" is related to "Money" in the same way as "Transport" is related to ? 1. Movement 2. Road 3. Speed 4. Traffic 5. Goods? The answer is 5) Goods. A bank deals with transaction of 'money', where as the 'transport' deals with the movement of 'goods' "Horse" is related to "Hoof" in the same way as "Eagle" is related to ? 1. Leg 2. Clutch 3. Claw 4. Foot 5. None of the above 1. The answer is 3) Claw. 1. The lower part of feet of "Horse" is know as "Hoof", where as lower part of feet of "Eagle" is called as "Claw"."Marriage" is to "Divorce" as "True" is to ? 1. Truth 2. Story

3. Fiction 4. False 5. None of the above The answer is 4) False "Divorce" is the antonym of "Marriage". In the same way "False" is the antonym of "True". Leaf is related to Sap in the same way as Bone is related to ? Marrow 2. Fluid 3. Blood 4. Calcium 5. None of the above The answer is 1) Marrow Sap is the fluid contained in Leaf in the same way marrow is the fluid contained in Bone. Which of the following pairs of words bears the same relationship as that of the pair of words given at the question place? Cells : Cytology :: Insects : 1. Ornithology 2. Entomology 3. Anatomy 4. Onomatology 5. None of the above The answer is 2) Entomology Cytology is the study of Cells and Entomology is the study of Insects. (Extra info : Ornithology is the study of Birds, Anatomy is the study of the structure of human or animal bodies and Onomatology is the study of Names) What is found once in Tea, Twice in Coffee, but not in milk? 1. Water 2. Cream 3. Sugar 4. "E" 5. None of the above The answer is 3) "E" The English alphabet "E" is found once in the word "Tea", twice in the word "Coffee" and there is no such alphabet in the word Milk" Always : Never :: 1. Frequently : Normally 2. Intermittently : Casually 3. Rarely : Casually 4. Constantly : Frequently 5. Often : Rarely The answer is 5) Often : rarely "Always" and "Never" are antonyms in the same way "Often" and "Rarely" are also antonyms. Bull is related to Draught in the same way Cow is related to ? 1. Milch 2. Livestock 3. Farm 4. Fodder 5. None of the above The answer is 1) Milch Bull is a draught animal (beast of burden) and cow is a milch animal (milk - yielding) A "Square" is related to "Cube" in the same way as "Circle" is related to 1. Sphere 2. Circumference 3. Diameter 4. Area 5. None of The above The answer is 1) Sphere A square is a two dimensional figure consisting of sides where as a cube is a three dimensional figure. Similarly, circle is a two dimensional figure and a sphere is a three dimensional figure . "Iron" is related to "Solid" in the same way as "Mercury" is related to ? 1. Vapour 2. Solid 3. Gas 4. Liquid 5. None of the above The answer is 4) Liquid Iron is found in solid state. Similarly, mercury is found in liquid state. "12" is related to "36" in the same way as "17" is related to ? 1. 34 2. 63 3. 68 4. 71 5. 51

The answer is 5) 51 Three times of 12 is equal to 36 (12 X =3 = 36). Similarly, three times of 17 is equal to 51 (17 X 3 = 51). "Horse" is related to Hay in the same way as Cow is related to ? 1. Milk 2. Leaves 3. Straw 4. Fodder 5. None of the above Hay is the food of horse. Similarly, Fodder is the food of Cow. Horse : Hoof :: Cat : ? 1. Horns 2. Wings 3. Legs 4. Mane 5. Paws The answer is 5) Paws Hoof is the name of the feet of horse. Similarly, Paws is the name of the feet of the Cat Swan is related to Cygnet in the same way as Fish is related to ? 1. Fry 2. Fawn 3. Fin 4. Foal 5. None of the above The answer is 1)Fry Here Fry means not the Fish Fry :P Fry is the name of the baby fish :) Cygnet is the young one of Swan in the same way as fry is the young one of Fish. Young is related to Old in the same way as wide is related to ? 1. Big 2. Insufficient 3. Long 4. Narrow 5. None of these The answer is d) Narrow The words in ach pair are antonyms of each other. Tempest is to storm as slim is to ? 1. Slender 2. Plump 3. Fat 4. Beautiful 5. Stout The answer is 1) Slender First is of higher intensity than the second To be frank there is NO relation between the first and second pics. But they are similar in shape. So, logically they are similar. Themain purpose of posting this picture here is,you should scan all options perfectly and think twice before answering the questions belong to Analogy. As the answer first strikes in your mind may not be correct and there might be another logic behind the question. Blood Relations - Reasoning blood relations shortcut methods for Bank Exams Cousin : Mother's or Father's Brother's or Sister's Son or Daughter (or) Parent's siblings' son or daughter (or) Uncle's or Aunt's son or daughter. Nephew : Brother's or Sister's son Niece : Brother's or Sister's daughter. Uncle : Father's or Mother's brother. Aunt : Father's or Mother's sister Father-in-Law : Spouse's father (or) Wife's or Husband's Father Mother-in-Law : Spouse's mother (or) Wife's or Husband's Mother Son-in-Law : Daughter's Husband Daughter-in-Law : Son's wife Brother-in-Law : Spouse's brother (or) Sister's husband Sister-in-Law : Spouse's sister (or) Brother's Wife Maternal : of or related to Mother Paternal : of or related to Father Siblings : brothers or sisters Spouse : Husband or Wife Note : Cousin is a COMMON GENDER, There are NO such words Cousin Brother / Cousin Sister Generations : -2 Generation : Grand father and Grand Mother (Maternal & Paternal)

-1 Generation : Mother, Father, Brother, Sister, Brother-in-Law, Sister-in-Law, Cousin +1 Generation : Son, Daughter, Son-in-Law, Daughter-in-Law, Nephew, Niece +2 Generation : Grand Sons and Grand Daughters POINTS TO REMEMBER : If the question is "How is A related to B". Then you must know the gender of A to answer the question Without knowing A's gender, you cannot determine the relation from A to B If a person is Uncle or Aunt to "A", A is that person's nephew (if A is Male) or niece (if A is female) Cousin is a common Gender (I mean, you can use this word for both Male and Female) Ex : A says to B, "you are the son of my grand father's only son". How is B related to A? How is A related to B? Ans : B is Brother of A. My grand father's only son = A's father (grand father's only son means only ONE son.. No other son and no daughter also) A is either brother or sister to B A and B are siblings. B is brother to A as we know the Gender of B. But we dont know the Gender of A. So we cant say whether A is Brother or Sister to B. Lets have a look at some practice problems : The mother of Ranbir is the only daughter of Neetu's father. How Neetu is related to Ranbir? Sister Mother Aunt Cant determine None of These Answer : Mother Explnation : Only daughter of Neetu's father = Neetu only. The mother of Ranbir is Neetu, so Neetu is the mother of Ranbir :D Amit is the son of Ajit's grand father's only daughter. How is Ajit's father related to Amit? Grand Father Uncle Father Data Inadequate None of these Answer : Father Explanation : Ajit's grand father's only daughter means Ajit's mother. Amit is the son of Ajit's mother. So, Amit and Ajit are Siblings. Ajit's father is Amith's father too Pointing to a boy Rekha said, "He is the son of my mother-in-law's only Child". How is the boy related to Rekha? Son Grand Son Nephew Brother Cant determined Ans : Son Explnation : My mother-in-law's only child = Rekha's mother-in-law's only child = Rekha's husband. The boy is the son of the Rekha's Husband. So obviuosly Rekha is the mother to that boy and that boy is the Son of Rekha. B is the husband of C. A is the sister of B. D is the sister of C. How is D related to B? Son Uncle Sister-in-Law Cant be determined Ans : Sister-in-Law

Explnation : B is Husband and C is his Wife. A is the sister of B, so A is the Sister-in-Law of C ( Husband's sister) D is the sister of C so, D is the Sister-in-Law of B (Wife's Sister) Pointing towards a girl, a Person said, "She is the only daugher of the only son of the wife of the father-in-law of my wife". How is the girl related to the Person? Niece Daughter Sister Daughter-in-Law Cant be Determined Ans : Daughter Explnation : Father-in-Law of my wife = Father in Law of the Person's Wife = Person's Father Wife of the Father-in-law of my wife = Wife of the Person's Father = Person's Mother. Only son of Person's Mother = That Person only (because Person's parents dont have any other child) So, she is the only daughter of Person as the Person does not have any other Children. Coded Relations Before going to read this post, Read Basics of Blood Relations Here . Coded Relations is another important topic for Bank Exams. You can easily get 5 marks with little logical thinking and concentration. It doesn't take much effort to solve these type of questions... All that you need is Concentration and lil practice to solve these type of questions 1. If 'A X B' means 'A is the Daughter of B'. 'A + B' means 'A is the husband of B' and 'A - B' means ' A is the sister of B', then what does ' M + N - P X Q' Mean? Sol : M + N - P X Q => M is the Husband of N, N is the sister of P and P is the Daughter of Q. N and P are sisters and the daughters of Q. Q is either mother or father of N and P. so M is SonIn-Law to Q. Have a look at the below image for more clarifications. Directions (2-3): Study the following information carefully and answer the questions given below. ' P X Q' means 'P is Sister of Q' ' P + Q' means 'P is mother of Q' 'P - Q' means 'P is father of Q 'P % Q' means 'P is brother of Q' 2. Which of the following represents 'A is nephew of B'? 1. A % B - C 2. C % B - A 3. C X B + A X D 4. C % B - A % D 5. None of the Above Sol : A is nephew of B means, A must be MALE. The symbols '-' and '%' indicate the male Gender. So, A must be followed by either '-' or '%'. So, checking of options 1 and 4 will be enough to solve this question. 1. A % B - C => A is brother of B and B is father of C. Here, A is C's paternal Uncle. 4. C % B - A % D => C is brother of B, B is father of A and A is brother of D. So here, A is son of B. So the answer is, 5. None of the Above 3. Which of the following represents 'W is Grand Father of H'?? 1. W + T - H 2. W % T - H 3. W X T + H 4. W % T + H 5. None of these Sol: W is Grand Father of H means W must be a Male. The symbols '-' and '%' indicate the Male Gender. So, W must be followed by Either '-' or '%'. So, Obviously we have to check the options 2 and 4. 2. W % T - H => W is brother of T and T is father of H. Here, W is paternal uncle to H.

3. W % T + H => W is brother of T and T is mother of H. W is maternal uncle of H. So the answer is, 5. None of these Directions (4-5) : Following questions are based on the information given below. 'P X Q' means 'P is father of Q' 'P - Q' means 'P is sister of Q' 'P + Q' means 'P is mother of Q' 'P%Q' means 'P is brother of Q' 4. Which of the following represents 'J is son of F'? 1. J % R - T X F 2. J + R - T X F 3. J % M - N X F 4. Cant Say 5. None of the above Sol: J is son of F means J must be a Male. So, J must be followed by the symbols 'X' or '%". We have to check options 1 or 3. 1. J % R - T X F => J is brother of R, R is sister of T and T is father of F. So, J is paternal uncle of F. 3. J % M - N X F => J brother of M, M is sister of N and N is father of F. Here, J is paternal uncle of F. 5. In the expression B + D X M % N, how is M related to B? 1. SOn 2. Grand Daughter 3. Grand Son 4. Grand Daughter or Grand Son 5. None of the above

Comparison Test of Reasoning Friends, Comparison Test is one of the promising areas of Reasoning Section of Banking and other Competitive Exams. We are all good at Comparisons but problem is we often confuse while doing these type of problems. In this post we will try to help you solving comparison problems easily with some shortcut techniques. As all of you know, comparison is expressed in phrases like "is greater than", "is less than", "is equal to" etc. We have mathematical symbols to express the above mentioned comparisons. Greater Than --------- > Less Than --------- < Equal to --------- = You can avoid the confusion by using these symbols while dealing with the test of comparison. It not only reduces the confusion and saves the time, but also helps your mind to pick the correct answer quickly. In my childhood I had a big confusion between the symbols > and <. Which one is used for

which. Then my uncle told me a technique to avoid confusion. That is., the bigger side represents the greater value. See the symbols carefully. Its just appear like an Arrow head / tip > The bigger side represents the greater number and the smaller side (tip) represents the smaller number. Now lets discuss some problems on Comparison Test. 1. Vikas is taller than Shyam but shorter than Umesh. Umesh is taller than Rajitha but shorter than Ganesh. Shyam is taller than Rajitha. Now say who is the shortest person in the group ? Solution : To save time just pic the first letters of the names to perform operations, Vikas is taller than Shyam, that means V > S Vikas is shorter than Umesh, that means V < U So, you can write them as U > V > S -----------(1) Umesh is taller than Rajitha, that means U > R Umesh is shorter than Ganesh, that means U < G So, you can write them as G > U > R ------- (2) Shyam is taller than Rajitha, that means S > R ------ (3) from 1, 2 and 3 you can say , G>U>V>S>R Here, R represents Rajitha. So, she is shorter. Note : To make the concept clear and easy to understand, we made these many steps with equations. But this is very simple process. Y ou just should take the decisions with a glance and should only write the final result on the paper. 2. Five of my friends wrote IBPS PO Exam. Alok got good rank than Suresh. Surech got good rank than Prakash. Alok's rank was not as good as Nikhil's. Kabir secured a rank between Alok and Suresh. Now who has more chances for the selection ? Solution : 1. A > S 2. S > P 3. A < N (or) N > A 4. K is in the middle of A and S. But from statement 1 we know that A is greater than S. So, A>K>S so finally , N > A > K > S > P So, Nikhil has good chances of selection. 3. In a group of 5 girls, Kamini is the second tallest girl. Pooja is taller than Mounika. Roopa is tallest among all. Neelam is taller than Pooja. If we make them stand in ascending order (according to their heights), who stands at the second position ? Solution : K is 2nd tallest P>M R > among all (so she is first, we already know that K is second) N>P So, R > K > N > P > M here K is in second position. So the answer is K, right? wrong. Read the question carefully . They 've mentioned that the girls stand in ascending order (that means short to tall). But above we have arranged them tall to short. So the answer will be P 4. A is taller than B, C is taller than D. D is taller than B. So who is the tallest person in the group ? Solution : According to the given data, A>B C>D D>B so, A > B and C > D > B here, we know that A is taller than B and C, D are also taller than B. But here A may be taller than C and D or shorter than Them. Or it may be between C and D. We can't make exact relation between the heights of A, C and D. So here we cant say the answer. 5. Amit and Sumit are twins. Richa is younger than Sumit. Richa is younger than Jyothi but older than Sourabh. Sumit is younger than Jyothi. Who is the oldest among all ? Solution : Here, there are two names starting with the same letter S (Sumit and Sourabh). So to avoid confusion treat Sumit as Su and Sourabh as So. A and S are twins, so A = Su

According to the given data, J > A = Su > R > So So, Jyothi is oldest among all. 6. A group of friends have gold coins. Ramu has a gold coin which is heavier than Mohan's and Valuable than Ramesh. The value of Naresh is more valuable than Ramu's. Naresh has a coin which is lighter than Yogesh. Mohan's coin is cheaper than Ramesh's. Who's coin is more valuable ? Solution : Note : Here you have two similar names Ramu and Ramesh. Both are starting with letters Ram. So here you can take them as Rm and Rh The main intention of the above problem is to make you confused. Some times they mentioned about weight and some times the value. But here the point is the value of the gold depends on its size. The more heavier the coin, the more value it gets. So, Rm > M Rm > Rh N>R Y>N Rh > M according to above equations, you can say Y > N > Rm > Rh > M So, the coin of Y ogesh is more valuable. 7 . P is heavier than T but lighter than M. N is lighter than S and T. Q is heavier than D but lighter than N. S is not heavier than M. Who is the heaviest among all ? Solution : Here we can arrange them in 3 way s, M > P > T > S > N > Q > D or M > P > S > T > N > Q > D or M>S>P>T>N>Q>D In all combinations, M is the heaviest among all Note : If they ask you about the second / third / fourth heaviest person in the group, you Can't say the answer. 8. Pooran has more bank balance than Sushma but lesser than Singh. If the bank balances of Pooran, Sushma and Singh are X, Y and Z, then which of the following equations is correct ? 1. X < Y < Z 2. Y < X < Z 3. Z < X < Y 4. X < Z < Y Solution : 2) Y < X < Z 9. Sita, Malathi, Reshma, Mary and Kamala are going on a safari. For every 5 Kilometers they planned to change the leader according to the Alphabetical Order. And planned to take a tea break for every 10 Kilometers. If they start with Kamala, then who will be the leader of the group after second tea break ? Solution : If we arrange their names in alphabetical order, K Mal Mar Re Si 12345 They start with Kamala as their leader. After 5 kilometers Malathi will be the leader. After 5 kilometers First Tea break (still Malathi is the leader) After 5 kilometers Mary will be the leader After 5 kilometers Second Tea break, (still Mary is the leader) so the answer is Mary . 10. Amar, Akbar, Antony and Peter are participating in a running race. Amar can run faster than Akbar, but cant run faster than Peter who cant run as fast as Antony. Now answer the following questions. 1. Who is the fastest runner among all ? 2. Who is slower among all ? 3. Who comes second in the race ? Solutions : According to the given data, An > Pe > Am > Ak 1. Antony 2. Akbar 3. Peter That's all for now friends. Y ou can read more reasoning shortcut techniques from here. Happy Reading :)

Verbal Reasoning Shortcuts - Direction Sence N sponsored links ow a days in almost all competitive exams you get questions which test your sense of direction in verbal reasoning section. These questions typically involve a person's moving certain distances in specified directions. Then, the student is asked to find out the net distance traveled between two points. The easiest way of

solving these problems is to draw a diagram you read information about the problem and let the diagram reflect all the information given in the problem. The distance from a particular point after traveling a distance of X meters in the horizontal direction and a distance of Y meters in the vertical direction is equal to X 2+y 2 (Please note that in common usage, north-south direction is referred to as "vertical" direction and east-west direction is referred to as "horizontal direction. To solve these types of problems, the student should know the directions properly - without any confusion. The following diagram shows all the directions and the student should memorize the diagram. If you have problems in remembering the above diagram then just rememb

as

er the word NEWS and a reverse S. Of course, this S is not like your Normal S. Its kinda kiddish. Have a look at the below figure. Start with N (for North) and move to E, W and finally S. Mark the corner of North and East as North-East, South and East as South-East and so on... Well, you have your own diagram now :P Now lets have discuss this topic with some examples. So that you can easily understand what exactly is Direction Sense and how to solve the problems. 1. A person travels towards east from his house and travels a distance of 3 meters; he then travels a distance of 7 meters south wards and then travels towards east a distance of 3 meters and finally travels southwards a distance of 10 meters. What is his vertical distance from his house? Solution : The distance traveled by him is equal, to 10 + 7 = 17m. Have a look at the below figure for

detailed understanding.

2. A person starts from his house and travels 5 meters towards east, then travels 6 metres towards right, then travels 8 meters towards east and travels 2 meters towards south after that. Finally he turns right and travels 7 meters. What is the total distance he has traveled from his house in the north-south direction?

Sol : The distance he traveled from his house in north-south direction is equal to 6 + 2 = 8 Meters

3. A person travels 7 meters towards east, then he turns right and travels 2 meters; then travels 5 meters towards left and then proceeds 2 meters northwards and finally travels 2 meters westwards. How far is he from his house in the vertical direction? Sol : The distance covered by the person in the north-south direction from his house is equal to 22 = 0 meters (Here please keep in mind that he has actually traveled 2+2 = 4 meters in the northsouth direction but, of that distance, since 2m is towards north and 2m towards south, effectively, he is 0m away from his house in the north-south direction). 4. Starting from one location, a person travels a distance of 5 meters southwards, then travels a distance of 7 meters leftwards, then travels 5 meters northwards and finally travels 6 meters eastward to reach a new location. What is the distance he travelled from his previous location? Sol : The distance traveled by vertically is 5-5 = 0 meters and the distance travelled horizontally is equal to 7+6 = 13m. Therefore, the distance travelled from his original location is also equal to 13m. 5. A person starts from his house and goes 2 meters towards east, then turns towards right and goes 25 meters and again goes towards east traveling 15 meters and then turns left and travels for 18 meters. He then goes towards

east and travels 7 meters. How far is he from his house? Sol : If we represent the path covered by him in a diagram, it will appear as follows. The total distance travelled horizontally is equal to 2+15+7 = 24 metres and the total distance travelled vertically is 25-18 = 7 metres.

So, the total distance travelled will be equal to root (24 2+7 2 ) = 25m That's all for now friends. We shall discuss some more practice exercises on Directions in our next post.All the best and happy reading :) Data Sufficiency - Introduction and Shortcuts D sponsored links ata Sufficiency is one of the hardest part of the Logical Reasoning section which involve very simple calculations. Of course, many of you may not agree with me with this point. But its fact, because in many of the cases negligence or over thinking makes you ending up losing marks. With little concentration and practice you can easily score good marks in this section. After all you don't even need to do complete calculations to get answer. Most of the times, with a simple glance on given options you can choose the correct answer. Of course there is no technical definition for this, but simply you can say that "its a process of checking whether the given data is sufficient to get the answer or not". Unfortunately there is no specific area, they can ask questions from any area like Reasoning, Arithmetic, Pure Maths, Time and Work, Time and Distance, Geometry, Menstruation, Percentages, Simple and Compound Interests etc.... So its better to prepare all areas (knowing basic formulas and procedures is enough. No need to be expertise. Now lets have a look at some example problems so that you will get a brief idea on Data Sufficiency problems and solutions. Directions for questions 1 to 10 : Each problem contains a question and two statements which give certain data. You have to select the correct answer from (1) to (4) depending on the sufficiency of the data given in the statements to answer the question. Mark 1 : If statement I alone is sufficient to answer the question and statement II alone is not sufficient to answer the question. Mark 2 : If statement II alone sufficient to answer the question and statement I alone is not sufficient to answer the question. Mark 3 : If statements I and II together are sufficient to answer the question, but neither statement alone is sufficient. Mark 4 : If statements I and II together are not sufficient to answer the question adn additional data specific to the problem are needed. 1. Which day of the week is 13th March of the year X ? 1. 25th December of the previous year is a Tuesday. 2. The previous year is a non-leap year. Solution : From Statement I alone, 25th December of the year preceding X is a Tuesday. But we do not know whether year X is a leap year or not. Hence, I alone is not sufficient. From Statement II alone, we don not know whether X is a leap year or a non-leap year. This information is not sufficient to answer the question. Hence II alone is not sufficient. From I and II together also we cannot know whether year X is a leap year or not. so you should choose option 4. 2. Is 2nd October of year Y a holiday ? 1. The third Sunday of July of year Y falls on 15th Day. 2. Only Sunday's are holidays. Solution : From statement I alone, 15th July is a Sunday. Hence it can be found out which day of the week is 2nd October. But that does not answer the question. Hence, I alone is not sufficient. From statement II alone, only Sunday's are holidays. But it cannot be determined whether 2nd October is a Sunday or not. From I and II together, it can be found out whether 2nd October is a Sunday or not and hence the question can be answered. So Option 3 3. Among five persons A, B, C, D, and E, sitting in a row (not necessarily in the same

order) who is at the middle of the row ? 1. A and E are at the extreme ends and C is to the right of A. 2. B is between C and D, neither of whom are at extreme ends. Solution : From I alone, A is at the extreme left and E is at the extreme right. C is at the 2nd, 3rd or 4th place from left. But, who occupies the 3rd place is not known. Hence I alone is not sufficient. From II alone, B is between C and D. Neither C nor D are at extreme ends. Hence C and D are in 2nd and 4th places in any order and B is in 3rd place. Hence II alone is sufficient. So option 2 4. Six persons P, Q, R, S, T and U are sitting around a circular table (not necessarily in the same order). Is P sitting opposite to T? 1. S is to the left of R and U is to the left of Q. 2. Q is sitting opposite to R. Solution : From I alone, if S, R, U and Q are sitting adjacent to each other, then P and T are not opposite to each other. If Q and R are opposite to each other then P and T will be opposite to each other. Hence, I alone is not sufficient. From II alone, Q and R are opposite to each other. This information is not sufficient to answer the question . Hence II alone is not sufficient. From I and II together, Only one arrangement is possible. From this we can conclude that P is

not sitting opposite to T. So option 3 5. What is the angle between the two hands of a clock ? 1. One hour ago, the angle between the two hands was 75 Degrees and the minute hand was ahead of the hours hand. 2. The hour hand is between 9 and 10. Solution : From I alone, in 60 minutes the minute hand reaches the same place and the hour hand moves forward by 30 Degrees. Hence, the angle reduces to 45 degrees. Hence I alone is sufficient. Option I 6. What is the time indicated by the clock ? 1. The minute hand is at 4. 2. The hour hand is past 9. Solution : From I alone, the position of the hour hand is not known. Hence, I alone is not sufficient. From II alone, we don not know whether the hour hand is past 10 or not and the position of minutes hand is not known. From I and II together, the exact position of hour hand is not known. The time could be 9:20, 10:20 or 11:20. So option 4 7. Is A brother of C ? 1. A's father's only sibling, X, is C' paternal aunt. 2. A is the daughter of B. Solution : From I alone, A's father's sibling 'X' is female. She is the aunt of both A and C. It is not know whether A is a male of Female. Hence, I alone is not sufficient. From II alone, A is female. Hence A is not the brother of C. Hence II alone is sufficient. So option 2 8. How is P related to T ? 1. T is the father of P. 2. T's son-in-law is P's brother-in-law.

Solution : From I alone, P is either the son or the daughter of T. Hence, I alone is not sufficient. From II alone, it is not know whether P is a male or a female. II alone is not sufficient. From I and II together, we cannot conclude whether P is a male or female. So option 4 9. Is C the tallest among A, B, C and D ? 1. A is as tall as B and D, but shorter than C. 2. B and D are shorter than C. Solution : From I alone, C is the tallest among A, B, C and D. So option 1 10. Is P the richest among L, M, N, O and P ? 1. M is the poorest. N and O are poorer than L. 2. No one is richer than L Solution : From I alone, L is the richest among L, M, N and O. No information regarding P is available. Hence I alone is not sufficient. From II alone, no one is richer than L. Hence, L is the richest. So option 2 That's all for now friends. In our next post we shall discuss more problems on Data Sufficiency. All the Best and Happy Reading :) Syllogism Shortcuts - Lesson 1 I sponsored links n almost all competitive exams, there will be a few questions on "Syllogism". In these type of questions they will give two or more statements followed by two or more conclusions. You have to choose which conclusion logically fits the given statements. The answer has to be chosen from the five given choices. Often these statements are funny and seem to be at variance of commonly known facts, but you should take them as they are True. It sounds very easier, but unfortunately most of the people tend to choose wrong answers by assuming them correct. In this post we shall discuss some shortcut techniques to perform well in syllogism section of competitive exams. Generally these questions can be answered by representing the given statements by Venn Diagrams. However, here we shall discuss another simple shortcut techniques to solve Syllogism problems in easy manner. Before going into details, lets look at some basic terms used in the rules and understand what they mean. The two statements is given in the question are called "Premises" and the answer, the "Conclusion". Eg : All Dogs are Cats ----------(1) All Cats are Pigs ------------(2) The above two statements are called "Premises". From above two statements, we can draw the Conclusion : All Dogs are Pigs. The premises normally start with the words All, No, Some, Many , Some -- not, Many -- not. These words are referred to as qualifiers. A premise consists of a subject and a predicate wherein the first term [eg. "Dogs" in statement (1)] is the subject and the second term [eg. "Cats" in statement (1)] the predicate. Similarly, intatement (2) "Cats" is called the subject and "Pigs" is called the predicate. The word that occurs in both the premises is known as the "Middle Term". ("Cat" in the example above). The answer or "Conclusion" should consist of the other two words (dogs and pigs in the example above) and the middle term should not appear in the answer. The premises can be divided into 1. Universal Statements 2. Particular Statements This classification of the premises into the above categories is dependent on the qualifier used in the premise. For example, statements where "All" is used are called Universal statements and statements where "some" is used are called Particular Statements.

Premises can also be divided into 1. Positive (affirmative) statements and 2. Negative statements If there is a negative term like "not" or "no" in the statement, it is called a negative premise. Otherwise it is called a positive premise or an affirmative statement. The combination of the two different categories of classifications leads to four different premises as given in the below table. Table I : Affirmative Negative Universal All A E Particular Some; many I

NO Some not; many not O

The subject or the predicate can either distributed or not distributed in the given premise. The Subject and predicate are either distributed () or not distributed ()depending on what kind of a statement it is (particular affirmative, etc.). Below table shows the distribution pattern of the subject and predicate. Table II : Subject Predicate Universal Affirmative Universal Negative Particular Affirmative

Particular Negative

Note : Indicates Distributed. Indicates Undistributed Syllogism Shortcuts with Examples - Lesson 2 I sponsored links n our Syllogism Shortcut Lesson 1, we have discussed about the Introduction of Syllogism. So before going to read this Lesson 2, it is advisable to read that Lesson 1 here. In this post we shall discuss some important rules with Examples. 1. Every deduction should contain three and only three terms. 2. The middle term must be distributed at least once in the premises. 3. If one premise is negative the conclusion must be negative. 4. If one premise is particular the conclusion must be particular. 5. If both premises are negative no conclusion can be drawn. 6. If both premises are particular no conclusion can be drawn. 7. No term can be distributed in the conclusion if it is not distributed in the premises. Now we shall discuss these rules with examples. Example 1 : All dogs are cats ---- (1) All cats are pigs ----- (2) As the first statement is a universal af irmative statement, the subject (dogs) has to be distributed () and the predicate (cats) not distributed (). As the second statement is also universal af irmative the subject cat is distributed () and the predicate pigs not distributed (). The above is arrived at on the basis of Table II. The middle term ("cats" is the middle term as it occurs in both the premises) is distributed once in the premises. Hence it satisfies Rule [2]. As "Dogs" is distributed in the premise and "Pigs"

undistributed, in the deduction also, they should appear accordingly. The type of statement that satisfies both of them is universal affirmative statement, i.e., a statement with "All". Hence the answer will be

All dogs are pigs The answer cannot be "All pigs are dogs" because Rule [7] states that no term can be distributed in the conclusion if it is not distributed in the premises. As "pigs" is not distributed in premise it cannot be distributed in the conclusion (because if we take "All pigs are dogs", then the subject "pigs" will be distributed). Hence the conclusion "All pigs are dogs" is wrong. Example II : All cats are dogs ---- (1) All cats are pigs ----- (2) Statement I is Universal affirmative and hence the subject "cats" is distributed and the predicate "dogs" is not distributed as per Table II. Statement II is also Universal affirmative and hence the subject "cats" is distributed and the predicate "pigs" is not distributed as per Table II. here the middle term "cats" ("Cats" is the middle term as it is occuring in both the premises) is distributed; hence we can draw a conclusion. The answer should contain the terms "dogs" and "pigs" and both the terms are not distributed. Referring to Table II, we find that this is possible only in Particular Affirmative [the conclusion cannot start with the qualifier "All" as the subject in "All" should be distributed]. According to Rule 7 a term cannot be distributed in the conclusion if it is not distributed in the premises. So the answer will be "Some dogs are Pigs" or "Some Pigs are Dogs" Example III : All Dogs are Cats ------ (1) All Pigs are Cats ------- (2) Statement (1) is universal affirmative and hence the subject "Dogs" is distributed and the predicate "Cats" is not distributed. In statement (2) which is also a universal affirmative, the subject "Pigs" is distributed and the predicate "Cats" is not distributed. This is arrived at on the basis of Table II. The middle term "Cats" ["cats" is the middle term as it occurs in both the statements] is not distributed in either one of the two statements. From Rule [2], which states that the middle term should be distributed at-least once in the premises for drawing a conclusion, we cannot draw any conclusion in this case. Example IV : All Cats are Dogs ------ (1) Some Cats are Pigs ------- (2) The first statement is a universal affirmative premise and hence the subject "cats" is distributed () and predicate "dogs" is undistributed (). The second statement is particular affirmative and hence both the subject "cats" and the predicate "pigs" are undistributed () as per Table II. As we have a particular premise, the conclusion should also be a particular one as per Rule [4]. The middle term is distributed hence we can draw a conclusion. So the answer will be Some Dogs are Pigs or Some Pigs are Dogs Example V : All Dogs are Cats ------ (1) No Cats are Pigs ------- (2)

As the first premise is universal affirmative the subject (dogs) is distributed and the predicated (cats) is undistributed. In the second premise which is universal negative the first term (cats) and the second term (pigs) are both distributed (as per Table II). As the middle term is distributed atleast once in the premises, Rule [2] is satisfied and hence we can draw a conclusion. From Rule [3] which states that if one of the premise is negative the conclusion should be negative, the answer should be a negative one and as both the terms dogs and pigs are distributed the conclusion should be a universal negative statement. Hence the answer will be No dogs are pigs or No pigs are dogs Example VI : All dogs are Cats ---------- (1) Some cats are not pigs --------- (2) Since the first statement is universal affirmative, "dogs" is distributed and "cats" is not distributed. Since the second statement is particular affirmative, "cats" is not distributed and "pigs" is also not distributed (as per Table II). In the above given example no conclusion can be drawn as Rule [2] which states that the middle term ("Cats" in the example above as it occurs in both the premises) should be distributed at least on in the premises not satisfied. Syllogisms Examples - Lesson 3 T sponsored links his is the 3rd lesson of our syllogism shortcut series. If you are new to this post then first read Lesson 1 Here and Lesson 2 Here before reading this post. In our last post we have covered 6 examples. Here we shall start from the 7th one. Example VII : All Dogs are Cats ---------- (1) Some Cats are not Pigs ------------ (2) The first statement is a universal affirmative and hence the subject (dogs) is distributed and the predicated (cats) is not distributed. The second statement is particular negative and hence the subject (cats) is not distributed and the predicated pigs is distributed (Table II). But as the middle term (Cats) is not distributed atleast once in the premises, Rule [2] is not satisfied and hence we cannot draw any conclusion.

Example VIII : All Cats are Dogs ------------- (1) Some Cats are not Pigs --------------- (2) The first statement is a universal affirmative and hence "cats" is distributed and "dogs" is not distributed. The second statement is particular negative and hence "cats" is not distributed and "pigs" is distributed (as per Table II). Here the middle term (cats) is distributed and hence we can draw a conclusion. The conclusion should be particular negative as Rule [3] states that if a premise is negative the conclusion should also be negative. Also Rule [4] states that if a premise is particular the conclusion should also be particular. Hence the conclusion should be particular negative In particular negative, we know that the subject is not distributed and the predicated distributed.

The terms "dogs" and "pigs" should come in the conclusion. Also, since "dogs" is not distributed in the premise, it cannot be distributed in the conclusion also as per Rule [7]. As per the above reasoning, "Pigs" can be only the subject in the conclusion and hence "Dogs" will be the predicate. Thus the answer will be - Some dogs are not pigs Example IX : No Dogs are Cats -----------(1) No Cats are Pigs ----------(2) We cannot draw any conclusion as Rule [5] states that if both the premises are negative we cannot draw any conclusion. Example X : No Dogs are Cats ----------- (1) Some Cats are not Pigs ---------- (2) As both the statements are negative as per Rule [5] we cannot draw any conclusion. (The first statement is universal negative and hence the subject (dogs) is distributed and the predicate (cats) is also distributed as per Table II. The second statement is particular negative and hence the subject is not distributed and the predicate (pigs) distributed as per Table II). Example XI : Some Cats are not Pigs ----------- (1) Some Cats are Dogs -------------(2) As the first statement is particular negative the subject (Cats) is undistributed and the predicate (Pigs) distributed. In the second premise both the subject and predicate (Cats and Dogs respectively) are not distributed since the premise is particular affirmative (as per Table II). No conclusion can be drawn as both the premises are particular as per Rule [6]. Example XII : Some Cats are not Dogs ----------------- (1) Some Cats are not Pigs ------------ (2) We cannot get an answer from the statements as Rule [5] states that if there are two negative statements no conclusion can be drawn. Also Rule [6] states that if there are two particular statements no conclusion can be drawn. That's all for now friends. In our next post we shall discuss another important Reasoning topic. Happy Reading :) Input-output Arrangements Shortcuts n today's post we shall discuss one of the most promising, easiest and time taking model of the Reasoning paper Input Output arrangements. People often leave this area as this is little time taking. But with little practice and concentration you can easily get 5 out of 5 marks in this area within less time. First of all lets see how the problem will look like. In these type of problems they will give you an input line of words and numbers and rearranges them in a few steps (normally 5 to 7 steps). and you should find out the logic behind the rearrangement and work with the problems given by them. Lets see an example. Directions : Study the following information carefully and answer the following questions : A word and number arrangement machine when given an input line of words and numbers rearranges them following a particular rule in each step. The following is an illustration of input and rearrangement : Input : Now 41 28 Credit Join 37 Go 61 Step 1 : 61 Now 41 28 Credit Join 37 Go Step 2 : 61 Credit Now 41 28 Join 37 Go Step 3 : 31 Credit 41 Now 28 Join 37 Go Step 4 : 61 Credit 41 Go 37 Now 28 Join Step 5 : 61 Credit 41 Go 37 Now 28 Join Step 6 : 61 Credit 41 Go 37 Now 28 Join Step 7 : 61 Credit 41 Go 37 Join 28 Now Step 7 is the last step for this input. As per the rules followed in the above steps, find out in each of the following Questions the appropriate step for the given input. (followed by some questions) Now first lets have a look at the given problem. The logic in the arrangement is : The input is the combination of words and numbers. Firstly, the numbers got arranged in descending order. Whereas the words get arranged in alphabetical order. Numbers occupy the odd places and words

occupy even places in the final step. When any element gets arranged, the previous element occupying that place shifts one place towards right. And one more basic rule here we have to remember is, we can make only one change in one step. In step One, 61 occupies the first place from the left end and the other elements are pushed one place rightward. Similarly, in the step 2 now occupies the second place from the left end and the other elements are pushed one place rightward. So, alternate arranging of numbers and words finally gives the last step in which the odd places from the left are occupied by numbers and the even places are occupied by the words. Shortcuts : 1. When ever you see this type of problems just see the last step first. So that you can understand the logic without wasting your valuable time (generally these arrangements will be of assenting and descending orders). Then just check from bottom to top for the arrangement of words. And then only check the questions. 2. If you cant get answers mentally, you should write them on paper, but its waste of time. In those type of situations just write the first letters of the words. So, for the above example you can write N 41 28 C J 37 G 61 and work with this example. Suppose you encounter two words with the same starting letter then you should write two letters instead of one. Ex : If you encounter Gun and Goat as two words you will be confused if you write two Gs. so just write Go and Gu to avoid confusion. 3. Without giving you the raw data if they give you some third or fourth step and ask you to find out the Sixth step there is no need to solve the problem completely. If the given step is the third one atleast three words will be already arranged in order. So just check the given arrangement and check how many words are arranged in order and just strike those words with pencil. And just work with the remaining words (as there is no need to change the already arranged words again). It will save your time and effort. 4. In some cases the required word or number which is to be arranged will be the first letter in the first letter of the resulting arrangement, in this case we will cut tha word or number but we will not increase the step counter as we do not have to shift it anywhere, it was already at its place 5. Keep in mind that If they give you an arrangement and asks you to guess the prior steps (I mean giving you the 5th or 6th step and asking you the prior steps 2nd or 4th), then the answer would be Cant be Determined. Because there wont be any rule for guessing backwards as the word may come from anywhere. Now lets have a look at some examples based on the above problem Here. Input - Output Arrangements - Examples I n last post we've discussed some shortcuts of the Input - Output arrangements problems of Reasoning section. In this post we shall work with some examples. Note : To save the space, We are not posting the previous example problem here. So please refer to the example problem here for reference. As per the rules followed in the above steps, find out in each of the following questions the appropriate step for the given input. 1 ) Input : Chair Wood 21 42 59 Height Bench 78. How many steps will be required to complete the rearrangement? 1. Three 2. Four 3. Five 4. Six 5. None of the above. Solution : write the first letters of the given problem. Step 1 : 78 C W 21 42 59 H B Step 2 : 78 B C W 21 42 59 H Step 3 : 78 B 59 C W 21 42 H Step 4 : 78 B 59 C 42 W 21 H

Step 5 : 78 B 59 C 42 H 21 W Step 6 : 78 B 59 C 42 H 21 W So, according to the logic, step 6 is the last step for this input (Numbers are in descending order and words are in alphabetical order in this step 6). So 6 Steps will be required to complete the rearrangement. So the answer is option 4. 2) Input : When You 22 Special 31 16 47 Town Which of the following steps will be the last but one? 1. 4th 2. 6th 3. 5th 4. 7th 5. Non of the above Solution : Step 1 : 47 W Y 22 S 31 16 T Step 2 : 47 S W Y 22 31 16 T Step 3 : 47 S 31 W Y 22 16 T Step 4 : 47 S 31 T W Y 22 16 Step 5 : 47 S 31 T 22 W Y 16 Step 6 : 47 S 31 T 22 W 16 Y So, the answer is Step 5. So option 3 is correct. 3) Step 4 of an input is : 74 Again 69 Call 17 32 Horse Desk Which of the following is definitely the input? 1. Again Call 74 69 17 32 Horse Desk 2. 74 Call Again 17 69 Horse 32 Desk 3. Call 74 Again 69 17 32 Desk Horse 4. Cannot be Determined 5. None of these Sol : As we have already discussed, this is the case of arrangement. So, the previous steps cannot be determined with certainty. We cannot estimate the exact input. 4) Step 3 of an input is : 82 Brown 74 Sugar Hobby Lady 32 49 Which of the following will be step 6? 1. 82 Brown 74 Hobby 49 Sugar Lady 32 2. 82 Brown 74 Hobby 49 Lady Sugar 32 3. 82 Brown Hobby 74 Hobby 49 Lady 32 Sugar 4. Cannot be determined 5. None of these. Sol : Step 3 is 82 Brown 74 Sugar Hobby Lady 32 49, And we can find that first 3 words are in a proper order. So just strike them off with pencil and work with remaining words. Step 4 : H S L 32 49 Step 5 : H 49 S L 32 Step 6 : H 49 L S 32 So, option 2 is correct. Note: If you don't have enough time and you want to take risk then just see the answers. They've given the step 3, so there are chances that the first 3 letters wont change (in this case). So obviously the answer will be either option 1 or option 2. But this method is not suggestible. 5) Input : Goal Team Ask 12 92 85 42 Sound. Which of the following will be the step 4 ? 1. 92 Ask 85 Goal 42 Sound 12 Team 2. 92 Ask 85 Goal 42 Sound Team 12 3. 92 Ask 85 Goal 42 Team 12 Sound 4. 92 Ask 85 Goal 12 42 Sound 5. None of These Sol : Input : G T A 12 92 85 42 S Step 1 : 92 G T A 12 85 42 S Step 2 : 92 A G T 12 85 42 S Step 3 : 92 A 85 G T 12 42 S Step 4 : 92 A 85 G 42 T 12 S So the answer is option 3. Thats all for now friends. Tomorrow we shall discuss some more problems of this type. Good Day and Happy Reading :) ALPHA-NUMERIC-SYMBOL TEST (Part-I) I sponsored links n this topic, a JUMBLED SEQUENCE of some Alphabets / Letters / Numbers / Symbols are given, followed by Certain Questions based on It. Now Lets See the BASICs of It. BASICS : Before starting with this topics lemme

clear some Meanings of the words "Following, Followed by, Preceding and Preceded by" Let me start with an example. Take Two consecutive alphabets (say A B), Here A is Preceding B <=> B is Preceded by A B is Following A <=> A is Followed by B Take One more Example. Now take 3 consecutive alphabets (say ABC), Here A is followed by B and C, But A is immediately followed by B, A is not immediately followed by C (Or We can say) A is not immediately followed by C A is followed by C, but NOT immediately. C is preceded by A and B, But C is immediately preceded by B, C is not immediately preceded by A (Or We can say) C is preceded by A but not Immediately Now Lets see some EXAMPLES How many such Numbers are there in the arrangement each of which is immediately followed by a Symbol and also immediately preceded by a Consonant? Sol : Check for the sequence of Consonant-Number-Symbol (Shortcut : just write the first letters CNS above the question and check for that arrangement) How many such Consonants are there in the arrangement each of which is immediately preceded by a Number and also immediately followed by another Consonant? Sol : Check for the sequence of Number Consonant - Consonant (Shortcut : Same as above. write NCC n check) How many such Symbols are there in the arrangement each of which is immediately preceded by a Number but NOT immediately followed by a Vowel? Sol: Check for the sequence of Number - Symbol - Not Vowel (Shortcut: Write NSV (Just strike the letter V (or u can put the TICK marks above the letters N and S) How many such Numbers are there in the arrangement each of which is not immediately followed by a Letter and not Immediately preceded by a Symbol? Sol: Check for the sequence of no Symbol - Number - no Letter (SNL) How many such Symbols are there in the arrangement each of which is Either Immediately preceded by a Number or immediately followed by a Consonant but not both? Sol: Check for the sequence of Number - Symbol / Symbol - Consonant (NS or SC) How many such Consonants are there in the arrangement each of which is either immediately preceded by a Number or immediately followed by a Symbol or Both? Sol: Check for the sequence of Number - Consonant / Consonant Symbol / Number - Consonant - Symbol (NC or CS or NCS) ALPHA-NUMERIC-SYMBOL TEST (Part - II) Read ALPHA-NUMERIC-SYMBOL TEST (Part-I) HERE Now Let Us discuss about the Sides: Left of Left ( - ) Left Left of Right (+) Right Right of Right ( - ) Right Right of Left (+) Left NOTE : If opposite sides (Left - Right) are given add up the numbers. If same sides (Left -Left or Right - Right) are given deduct Small number from the Big one.

Ex : Check the following Alphabetical Series ABCDEFGHIJKLMNOPQRSTUVWXYZ What is the 7th alphabet to the left of the 10th alphabet from the right end in the above series? Ans : 7th left - 10th right(7+10) right17th right. i.e., "J" (10th alphabet from the right end is Q and 7th alphabet to the left of 'Q' is 'J') What is the 5th alphabet to the left of the 6th alphabet from the left end in the above series? 5th left - 6th left (6-5) left 1st left 'A' What is the 2nd alphabet to the right of the 5th alphabet from the left end in the above series? 2nd right - 5th left (2+5) left7th left 'G' What is the 13th alphabet to the left of the 13th alphabet from the right end in the above series? 13th left - 13th right (13+13) right 26th right 'A' What is the 3rd alphabet to the right of the 16th alphabet from the right end in the above series? 3rd right - 16th right (16-3) right 13th right 'N' Problems on Alpha - Numeric Symbol Test READ THE DETAILED EXPLANATION HERE, BEFORE DOING PROBLEMS -> Study the following information carefully to answer these questions A8B6#7HU%3$FVR2I@141WE9^L5 1. If all the symbols are dropped from the above arrangement which of the following will be eigth from the left end ? 1. 3 2. $ 3. F 4. U 5. None of These Ans. 1) 3 Explanation : The element Highlighted in the Red Color in the following sequence is the 8th element from the left end after dropping all the symbols (#, %, $, @, 1 and ^) A8B67HU3FVR2I41WE9L5 2. Which of the following is Seventh to the right of fifteenth from the right end? 1.4 2. T 3. 1 4. w 5. None of These Ans. 1) 4 Explanation : 7th right of the 15th from the right end ---> (15-7) right --> 8th right element. (Please read our Tutorial here) The element Highlighted in the red color in the following sequence is the 8th element from the right end. A8B6#7HU%3$FVR2I@1!41WE9^L5 3. Four of the following five are alike in a certain way on the basis of their positions in the above arrangement and so form a group. Which is one that does not belong to the group? 1. 95L 2. I41 3. F2R 4. B73 5. None of These Ans. 5. None of These Explanation : Four Groups belong to the same group. So, there is no group which is the odd

man out 4. How many such numbers are there in the above arrangement each of which is immediately followed by a vowel but not immediately preceded by a number ? 1. None 2. One 3. Two 4. Three 5. More than Three Ans. 2) One Explanation : Not a Number - Number - Vowel --- > NNV The elements in Highlighted in the Red color are such numbers A8B637HU%3$FVR2I@141WE9^L5 5. How many such symbols are there in the above arrangement, each of which is immediately followed by a consonant and also immediately preceded by a number? 1. None 2. One 3. Two 4. Three 5. More than Three Ans. 3 ) Two Explanation : Smbol - Consonant ----- > SC

The elements Highlighted in Red color in the following sequence are such symbols A8B6#7HU%3$FVR2I@141WE9^L5 How to solve Seating Arrangement Problems ? S sponsored links eating Arrangement or Seating Plan is just a diagram or a set of writteninstructions that determines where people should take their seats. This is one of the crucial and promising sections of banks and other competitive exams. But as we have already discussed in our earlier post, even a single mistake can ruin your entire answer in result ending up with losing 5 marks allotted to this area. So we have to be little cautious while doing seating arrangement problems. Seating Arrangement is not any new word which directly invented / introduced for competitive exams. Its just picked up from our daily life. We follow this strategy in formal dinners, weddings, theaters, trains etc to avoid chaos and confusion upon entrance. But in competitive exams we should arrange given people in a specific shape. Most widely asked shapes are Circular, Octagonal, Square, Rectangle and Two or Three parallel lines. Generally as soon as seeing the question, people tend to draw the shape mentioned in the question. And latter put dots or lines according to the given details. Its not good practice I must say . Because it ruins your valuable time and often leads to extra confusion. So better use straight lines instead of circles and shapes. Drawing straight lines instead of circles also make it easier to find who is sitting opposite to whom. Lets have a look at below diagram for detailed understanding.

Ex : Assume that you have been asked to draw a circular seating arrangement of six people you can follow the above mentioned lines strategy for all shapes either it may be square, pentagon, hexagon etc. Now try to remember some important points : If A is sitting immediate left of B, that means B is on immediate right of A. To avoid confusion of left and rights in circular and other shapes' seating arrangement problems, you should assume that all people are facing to center (unless and until they specify the direction). Its alway s better idea to assume you are one among them so that it will be easier for you to get an idea of the arrangement. If you are unable to get the idea from a line then better skip that line and go to next line. Y ou can revisit to the skipped line after getting another clue. now lets have a look at some problems on seating arrangement so that you will get the concept clear.

Note : In seating arrangement problems 1. Six persons A, B, C, D, E and F are sitting around a circular table. B is sitting to the immediate right of E. C is not sitting next to F. F is sitting opposite to B and D is sitting opposite to E. Who is sitting to the immediate left of D ? Solution : Its given that there are 6 persons sitting around the table. Six persons that means you should represent 6 points. That means 3 lines. Draw 3 lines as shown in the below figure. Later it is given that B is sitting to the immediate right of E. So we shall start with E. Just choose any of the end of the line and mark it as E. And B is sitting immediate right to E. So put B at the right side of E. Now the arrangement will be look

like this, Later its given that C is not sitting next to F. We don't have any idea about this statement at present. So skip to next statement, i.e., F is sitting opposite to B. We know where B is. So mark F to the opposite to B. Its also given that D is sitting opposite to E. So mark D as opposite to E. Now the arrangement will

look like, So only A and C are available.... Now go to the previous statement which we have skipped. C is not sitting next to F. With this statement its clearly known that we cant place C besides F. So we can place A there. And now there is only one gap between D and B. We can place C there. So the final arrangement

of seating will be appear like this. Now lets have a look at another problem. 2. Eight persons P, Q, R, S, T, U, V and W are sitting around a circular table. 'S' is to the immediate right of W. 'V' is not next to either R or T. 'W' is to the immediate right of T, who is sitting opposite to R. U and W are sitting opposite to each other. Followed by some questions... Solution : Here there are 8 persons. That means 4 lines. Given that S sat immediate right of W. So start with W. And place S immediate right to W. 'V' is not next to either R or T.. No clue about this statement. So skip it. 'W' is to the immediate right of T, who is sitting opposite to R.. We know where W is... Given that it is immediate right of T. That means T should be immediate left of W. Now from the above statement we came to know that T is sitting opposite to R. So place R opposite to T. Now the arrangement will appear

like this... No check the skipped statement. 'V' is not next to either R or T. That means, we cant place V neither on both vacant sides of R nor at the single side of T. So the only vacant place remaining was opposite to S. U and W are sitting opposite to each other. So just place U opposite to W. Now there are 2 vacancies and two people (P and Q) are pending. There is no specific clue to find out their exact places. So there are equal chances for them to place any of the side. Usually they leave blanks like this to make you confused. In these cases its better to write P/Q (P or Q) at both the sides. In-fact they wont ask questions about P/Q because they left them clueless. So finally the arrangement will appear like this.

Now lets have a look at some questions based on above arrangement. 1. Who is sitting opposite to 'S' ? 1. V 2. R 3. T 4. U 5. None of these 2. Who is sitting two places to the right of 'W' ? 1. P 2. S 3. V 4. Can't Say 5. None of these 3. If R and V interchange their places, then who is opposite to 'T' ? 1. P 2. S 3. R 4. V 5. None of these 4. If V is to the immediate left of 'P', then who is to the immediate left of 'R' ? 1. U 2. P 3. Q 4. Can't Say 5. None of these That's all for now Friends. In our next post we shall discuss some more practice problems on seating arrangement. Good Day :) Seating Arrangement Practice Problems with Explanations for Bank Exams - Reasoning S sponsored links eating Arrangement (or Sitting Arrangement as some of us pronounce it) is one of the important topics of the Reasoning section of Bank Exams. The best thing here is, with little concentration and efforts you can get 5 out of 5 (and ofcourse 15 out of 15 in the case of IBPS CWE sometimes) marks easily. And the main disadvantage of these sort of problems is..... even a single mistake can ruin the entire solutions and it may results losing all the marks!!! So its always advisable to check your answers twice before coming to a conclusion and marking the answers. Here we are giving some practice problems of Seating Arrangement problems of the Reasoning Section. Just see the problems and try to solve them on your own. Later you can check with the answers

given below with complete solutions. You can check more reasoning related problems with explanations here. Good Day and Happy Reading :) Study the following information carefully and answer the questions given below: P, Q, R, S, T, V and J are sitting around a circle facing the centre. S is not an immediate neighbour of V. S is second to the right of T, who is second to the right of Q. R is third to the right of J and second to the left of P. 1. Who is on the immediate right of Q? (a) S (b) R (c) V (d) data inadequate (e) none of these 2. What is Js position with respect to P? (a) Third to the right (b) Second to the left (c) Second to the right (d) Data inadequate (e) None of these 3. How many of them are there between Q and S? (a) 2 only (b) 3 only (c) 4 only (d) 2 or 3 only (e) None of these 4. Who among the following is sitting between V and R? Seating ArrangementPracticeProblems withExplanations for BankExams - Reasoning - Gr8AmbitionZ www.gr8ambitionz.com/2012/09/seating-arrangement-practice-problems.html 2/10 (a) Q (b) J (c) T (d) S (e) None of these Answers: Here, you should draw a rough sketch with the given details... Your sketch will be similar to the

below diagram. 1. (b) 2. (c) 3. (d) 4. (a) Eight members A, B, C, D, E, F, G and H belonging to three families X, Y, Z go for weekend outing in three different cars I, II, III. Four out of the eight members are females. Members of any one family travel in different cars. Each car has at least one male and one female member. Each family has at least two members. A belongs to family Y and he travels in car III. D is wife of E and they travel in cars I and II respectively. H is son of B, who is wife of G, and they belong to family Z. C is daughter of F, who is wife of A. C travels in car II. G does not travel with F. 1. Which of the following groups of persons travels in car I? (a) D, F, G (b) D, E, G (c) D, G, H (d) D, F, H (e) None of these 2. Which car has only two members traveling in it? (a) I (b) II (c) III (d) II or III (e) Cannot be determined 3. Which of the following members of families Y and Z travel in different cars? (a) F,G (b) C,G (c) F,H (d) C,F (e) None of these

4. Which of the following groups of persons is a group of all females? (a) B,D,G (b) A,B,C (c) B,E,F (d) D,E,F (e) None of these 5. Which of the following members of families X and Y travel in the same car (a) C,F (b) D,F (c) C,D (d) F,E (e) None of these Answers: Cars family X family Y family Z I D (female) F (female) H (male) II E (male) C (female) G (male) III A (male) B (female) 1. (d) 2. (c) 3. (a) 4. (e) 5. (b) All the six members of a family A, B, C, D, E and F are traveling together. B is the son of C. But C is not the mother of B. A and C are a married couple. E is the brother of C. D is the daughter of A. F is the brother of B. 1. How many male members are there in a family? (a) 1 (b) 3 (c) 2 (d) 4 (e) 5 2. Who is the mother of B? (a) D (b) F (c) E (d) A (e) None of these 3. How many children does A have? (a) one (b) two (c) three (d) four (e) None of these 4. Who is the wife of E? (a) A (b) F (c) B (d) Cant be determined (e) None of these 5. Which of the following is a pair of females? (a) AE (b) BD (c) DF (d) AD (e) None of these Answers: 1. (d) 2. (d) 3. (c) 4. (d) 5. (d) (i) Five boys Abdul, Durgesh, Eshwar, Chand and Bhaskar and five girls Pallavi, Kavitha, Radhika, Savithri and Vasantha sit in two rows facing towards each other. All the boys are in one row and all the girls in the other row. (ii) Eshwar who is to the immediate right of Bhaskar and opposite to Pallavi is not at any end. Radhika, who is immediate to the right of Kavitha and opposite to Chand, is at one of the ends. (iii) Abdul is opposite to Kavitha who is the third to the right of Savithri. Durgesh and Vasantha are not opposite each other. 1. Who is in the middle of the row of boys? (a) Abdul (b) Bhaskar (c) Durgesh (d) Data inadequate (e) None of these 2. Who is immediately to the right of Pallavi? (a) Vasantha (b) Kavitha (c) Savithri (d) Data inadequate (e) None of these 3. Who is opposite to Durgesh? (a) Vasantha (b) Radhika (c) Pallavi (d) Data inadequate (e) None of these 4. How many boys are there between Abdul and Bhaskar? (a) Nil (b) 1 (c) 2 (d) Data inadequate (e) None of these Answers: Durgesh, Eshwar, Bhaskar, Abdul, Chand Savithri, Pallavi, Vasantha, Kavitha, Radhika 1. (b) 2. (a) 3. (e) 4. (a) Ranking Test I n this type of problems, generally the ranks of a person both from the top and from the bottom are mentioned and the total number of persons is asked. Now a days they are questions in the form of a puzzles of interchanging seats by two persons. Lets have a look at some models for a clear understanding of the topic. Amala is older than Mona and Rama. Anju is older than Mona but younger than Sunny. Rama is older than Sunny. Who among them is oldest? A) Sunny B) Mona C) Amala D) Anju E) None of these Ans: C. Explanation : Amala > Mona, Raman ... (Treat this as (I)) ;

Sunny > Mona ... (Treat this as (II)); Rama > Sunny... (Treat this as (III)); So, combining all, we get Amala > Rama > Sunny > Anju > Mona. If Anju is taller than Shilpa but shorter than Manju and Shilpa is as tall as Sarayu but taller than Shubhashini, then Sarayu is A) taller than Manju B) shorter than Anju C) as tall as Anju D) shorter than Shubhashini E) shorter than Shilpa Ans : B. Explanation : Manju > Anju > Shilpa = Sarayu > Shubhashini. In a unit test Pavan's marks in Mathematics is more than her marks in History but not more than Geography. Her marks in Science is more than that in History but not as much as in Mathematics and her marks in English is not as much as in Geography. In which of the given subjects did she score the lowest? A) History B) English C) Science D) Data inadequate E) None of these Ans: D. Explanation : Geography > Maths > History ... (i); Maths > Science > History ... (ii); Geography > English ... (iii) Thus the lowest score was either in History or in English Ten boys are standing in a row facing the same direction. Dayaram, who is seventh from the left end of the row, is to the immediate right of Chaitu, who is fifth from the right end of the row. Chaitu is third to the right of Vipul. How many children are there between Dayaram and Vipul? A) One B) Two C) Three D) Data inadequate E) None of These Ans: C. Fifteen children are standing in a row facing north. Ravi is to the immediate left of Hritik and is eighth from the left end. Arjun is second from the right end. Which of the following statement is not true? A) Hritik is 7th from right end. B) There are four children between Hritik and Arjun. C) There are five children between Ravi and Arjun. D) Arjun is 13th from the left end. E) Ravi is exactly in the middle. Ans: D. P, Q, R, S, and T are sitting around a circular table. R is to the right of P and is second to the left of S. T is not between P and S. Who is second to the left of R? A) S B) T C) Q D) Data inadequate E) None of these Ans: C. In a row of boys Akash is fifth form the left and Nikhil is eleventh from the right. If Akash is twenty-fifth from the right then how many boys are there between Akash and Nikhil? A) 14 B) 13 C) 15 D) 12 E) None of these Ans: B. Explanation : There are (25-11-1) =13 boys between Akash and Nikhil. Haresh is 27th from the left end of a row of 49 boys and Mahesh is 27th from the right end in the same row. How many boys are between them in the row? A) 6 B) 3 C) 5 D) Data inadequate E) None of these Ans: B. Explanation : Since Harish is 27th from the left, then he is 49-27+1 from the right, i.e. 23rd from the right. Again, since Mahesh is 27th from the right, therefore there are three boys between them in the row. Arun is fifth from the left end and Navin is twelfth from right end in a row of children. If Navin shifts by three places towards Arun he becomes tenth from the left end. How many children are there in the row? A) 21 B) 22 C) 23 D) 24 E) None of these Ans: D. Explanation : 9 - D, Total number of children = 12+3+10-1= 24. If Nishanth is eleventh from the left in a row of boys, Rehman is fourteenth from the right, how many boys are there in the row? A) 25 B) 23 C) 36 D) Data inadequate E) None of these

Ans: D. Explanation : We do not have information regarding the number of persons between Nishanth and Rehman. In a row of girls, Ravina is 15th from the left and Mohini is 18th from the right. If they interchange their places, Mohini becomes 15th from the left. How many girls are there in the row? A) 33 B) 48 C) 47 D) Data inadequate E) None of these Ans: D. Explanation : We still need some more information. In a row of boys facing north, Sudhanshu is twelfth from the left. When shifted to his right by four places, he becomes eighteenth from the right end of the row. How many boys are there in the row? A) 32 B) 33 C) 34 D) Data inadequate E) None of these Ans: B. Explanation : 12+3+18 = 33. In a queue, Amrita is 10th from the front while Mukul is 25th from behind and Mamta is just in the middle of the two. If there be 50 persons in the queue, what position does Mamta occupy from the front? A) 20th B) 19th C) 18th D) 17th E) None of these Ans: C. Explanation : Number of persons between Amrita and Mukul = 50(10+25) = 15. Since, Mamta lies in middle of these 15 persons, so Mamta's position is 8th from Amrita i.e., 18th from the front. Raman ranks sixteenth from the top and forty ninth from the bottom in a class. How many students are there in the class? A) 64 B) 65 C) 66 D) Cannot be determined E) None of these Ans: A. Explanation : Clearly, number of students in the class = (15+1+48) = 64 Sanjeev ranks seventh from the top and twenty eighth from the bottom in a class. How many students are there in the class? A) 37 B) 36 C) 35 D) 34 E) None of these Ans: D. Explanation : Clearly, number of students in the class = (6+1+27) = 34. If Atul finds that he is twelfth from the right in a line of boys and fourth from the left, how many boys should be added to the line such that there are 28 boys in the line? A) 12 B) 13 C) 14 D) 20 E) None of these Ans: B. Explanation : Clearly, number of students in the line = (11+1+3) = 15. Number of boys tobe added = 28 - 15 = 13. Manisha ranked sixteenth from the top and twenty ninth from the bottom among those who passed an examination. Six students did not participate in the competition and five failed in it. How many students were there in the class? A) 40 B) 44 C) 50 D) 55 E) None of these Ans: D. Explanation : Number of boys who passed = (15+1+28) = 44. Total number of boys in the class = 44+6+5 = 55. Aruna ranks twelfth in a class of forty-six. What will be her rank from the last? A) 33 B) 34 C) 35 D) 37 E) None of these Ans: C. Explanation : Number of students behind Aruna in a rank = (46-12) = 34. So Aruna is 35th from the last. In a class of 60, where girls are twice of boys, Kamal ranked seventeenth from the top. If there are 9 girls ahead of Kamal, how many boys are after him in rank? A) 3 B) 7 C) 12 D) 23 E) None of these Ans: C. In a row of ten boys, when Rohit was shifted by two places towards the left, he became seventh from the left end. What was his earlier position from the right end of the row? A) First B) Second C) Fourth D) Sixth E) None of

these Ans: B. Classification for Meaningful Words - Practice Problems I sponsored links n this post, we shall discuss some examples of Classification for Meaningful Words. Four of the following five are alike in a certain way and so form a group. Which is the one that does not belong to that group? 1. Scientist 2. Engineer 3. Architect 4. Teacher 5. Doctor Show Answer Show Hint Four of the following five are alike in a certain way and so form a group. Which is the one that does not belong to that group? 1. Listen 2. Feel 3. Sing 4. Think 5. Hear Show Answer Show Hint Four of the following five are alike in a certain way and so form a group. Which is the one that does not belong to that group? 1. Anxiety 2. Feeling 3. Sorrow 4. Joy 5. Anger Show Answer Show Hint Four of the following five words are alike in a certain way and so form a group. Which is the one that does not belong to that group? 1. Three 2. Four 3. Five 4. Six 5. Nine Show Answer Show Hint Four of he following five are alike in a certain way and so form a group. Which is the one that does not belong to that group? 1. Anxiety 2. Worry 3. Inhibition 4. Curiosity 5. Weariness Show Answer Show Hint Four of the following five are alike in a certain way and hence form a group. Find the one which is different from the other four. 1. Rice 2. Wheat 3. Barley 4. Mustard 5. Bajra Show Answer Show Hint Four of the following five are alike in a certain way and hence form a group. Find the one which is different from the other four. 1. Arrow 2. Sword

3. Knife 4. Axe 5. Pistol Show Answer Show Hint Four of the following five are alike in a certain way and so form a group. Which is the one that does not belong to the group? 1. Pear 2. Jackfruit 3. Watermelon 4. Papaya 5. Mango Show Answer Show Hint Four of the following five are alike in a certain way and so form a group. Which is the one that does not belong to that group? 1. Pencil 2. Sharpener 3. Blackboard 4. Chalk 5. Pen Show Answer Show Hint Four of the following five are alike in a certain way and so form a group. Which is the one that does not belong to that group? 1. Tortoise 2. Frog 3. Rat 4. Mongoose 5. Snake Show Answer Show Hint Four of the following five are alike in a certain way and so form a group. Which is the one that does not belong to that group? 1. Swan 2. Crocodile 3. Frog 4. Snake 5. Chicken Show Answer Show Hint Four of the following five are alike in a certain way and so form a group. Which is the one that does not belong to that group? 1. Coriander 2. Potato 3. Beetroot 4. Onion 5. Ginger Show Answer Show Hint Four of the following five are alike in a certain way and so form a group. Which is the one that does not belong to that group? 1. Crow 2. Vulture 3. Bat 4. Ostrich 5. Eagle Four of the following five are alike in a certain way and so form a group. Which is the one that does not belong to that group? 1. Volume 2. Size 3. Large 4. Shape 5. Weight Show Answer Show Hint Four of the following five are alike in a certain way and so form a group. Which is the

one that does not belong to that group? 1. August 2. December 3. July 4. January 5. May Show Answer Show Hint Four of the following five are alike in a certain way and so form a group. Which is the one that does not belong to that group? 1. Jackal 2. heetah 3. Tiger 4. Lion 5. Dog Show Answer Show Hint Four of the following five are alike in a certain way and so form a group. Which is the one that does not belong to that group? 1. Cheese 2. Butter 3. Milk 4. Curd 5. Ghee Show Answer Show Hint Four of the following five are alike in a certain way and so form a group. Which is the one that does not belong to that group? 1. Sultry 2. Hot 3. Humid 4. Warm 5. Cool Show Answer Show Hint Four of the following five are alike in a certain way and so form a group. Which is the one that does not belong to that group? 1. Now 2. After 3. Then 4. Before 5. Again Show Answer Show Hint Four of the following five are alike in a certain way and so form a group. Which is the one that does not belong to that group? 1. Apple 2. Banana 3. Peach 4. Fig 5. Turnip Show Answer Show Hint Classification for Non-meaningful Words - Practice Problems sponsored links Directions : In each of the following questions, four groups of letters are given. Three of them are alike in a certain way while one is different. Choose the odd one. 1) RNJ 2) XTP 3) MIE 4) ZWR Show Answer Show Hint 1) PUT 2) END 3) OWL 4) ARM Show Answer Show Hint 1) EBD 2) IFH 3) QNO 4) YVX

Show Answer Show Hint 1) ALMZ 2) BTUY 3) CPQX 4) DEFY Show Answer Show Hint 1) STUA 2) RQPA 3) MLKA 4) HGFA Show Answer Show Hint 1) EDKL 2) LMST 3) NMUV 4) QPRS Show Answer Show Hint 1) XGEZ 2) PCAQ 3) LKIN 4) DWUF Show Answer Show Hint 1) PRVX 2) MQTV 3) DHKM 4) BFIK 2) Show Answer Show Hint 3) 1) BDYW 2) CEXZ 3) DFYW 4) EGXV 4) Show Answer Show Hint 5) 1) XZCG 2) OQTX 3) IMNQ 4) EGJN 6) Show Answer Show Hint 7) 1) UAZF 2) SCXH 3) RDWJ 4) KBPG 8) Show Answer Show Hint 9) 1) DFCE 2) HIGJ 3) NPMO 4) XZWY 10) Show Answer Show Hint 11) 1) ABCD 2) EGIK 3) ACDF 4) CFIL 12) Show Answer Show Hint 13) 1) RSXY 2) NOUV 3) MNST 4) DEJK 14) 1) xXYA 2) IIMP 3) hHIK 4) bBCE 15) Show Answer Show Hin Statement - Assumptions T oday we shall discuss about one more important topic of Reasoning part of Bank Exams, Statement - Assumptions. In these type of questions, they will give you a statement followed by two assumptions. You have to read the statement and assumptions clearly and find out an Implicit one. Lets see some examples. Read the following Statement and Assumptions. Give answer (a) if only assumption 1 is implicit. Give answer (b) if only assumption 2 is implicit. Give answer (c)if either assumptions 1 or 2 is implicit. Give answer (d) if neither assumption 1 not 2 are implicit. Give answer (e) if both assumptions 1 and 2 are implicit. 1) Statement : Vitamin E tablets improve circulation, keep your complexion in a glowing condition. Assumptions : 1) People like a glowing complexion. 2) Complexion becomes dull in the absence of circulation. Answer : e Explanation : Both assumptions are implicit. Generally, only those good features of a product are highlighted which people crave for. So, 1 is implicit. If circulation is improved, complexion grows. Hence, 2 is also implicit. 2) Statement : "The function will start at 6 pm. "You are requested to take your seats before 6 pm. Assumptions : 1) Function will start as scheduled. 2) If an invitee is not in his seat before 6 pm, the function will not start. Answer : a Explanation : It is mentioned in the invitation that you are requested to take your seats before 6 pm. It means that function will start as scheduled. Hence, assumption 1 is implicit. It is not given that the function will not start if invitees do not come in-time. 3) Statement : The X passenger car manufacturing company announced a sharp reduction in the prices of their luxury cars. Assumptions : 1) There may be an increase in the sale of their luxury cars. 2) The other such car manufacturers may also reduce their prices. Answer : a Explanation : The price of any product is lowered assuming that its demand will increase. Therefore, assumption 1 is implicit. There is no information about the other manufacturers, so assumption 2 is not implicit. 4) Statement : "if you want to give any advertisement, give it in the newspaper A" - X tells Y. Assumptions : 1) Y wants to publicize his products.

2) Newspaper A has a wide circulation. Answer : b Explanation : The word 'If' in the statement shows that Y may or may not want to publicize his products. So, 1 is not implicit. X advised that advertisements be given in news paper A. This means that A will help advertise better i.e. it has wider circulation. So, 2 is implicit. 5) Statement : Patient's condition would improve after this operation. Assumptions : 1) The patient can be operated upon in this condition. 2) The patient can't be operated upon in this condition. Answer : a Explanation : It is very much implied in the statement that the patient is in a position to be operated upon. Therefore, assumption 1 is implicit. But, assumption 2 is not implicit as it is just opposite to assumption. 6) Statement : The government has decided to allow the shopping complexes to remain open till midnight to reduce crowding of shoppers in these complexes during the weekends. Assumptions : 1) The sales of these shopping complexes may increase generating revenue for the govt. 2) People may still prefer to shop during the weekends. Answer : d Explanation : The objective of Govt. behind taking this decision is to facilitate people to shop at their leisure fimes during the late evening hours and not to increase its revenue. Hence, 1 is not implicit. No such information like people are interested to shop during only weekends. So, 2 is also not implicit. 7) Statement : The state government has decided to appoint thousand primary school teachers during next financial year. Assumptions : 1) There are enough schools in the state to accommodate four thousand primary school teachers during the next financial year. 2) The eligible candidates may not be interested to apply as the Govt. may not finally appoint such a large number of primary school teachers. Answer : a Explanation : As per the requirement of teachers in primary school only, the Govt. may have taken such decision. So, 1 is implicit. Assumption 2 is absurd as it is against the Govt's decission.

Coding - Decoding of Reasoning - Introduction F sponsored links riends, today we shall discuss about the Coding and Decoding type questions of Reasoning section. There are different types of Coding and Decoding questions are there, before going into details lets first understand what is meant by coding and decoding. Coding : A particular code or pattern is used to express a word in English language to express it as a different word. The coded word itself does not make any sense unless we know the code, i.e., unless we know the pattern or code that has been followed. Decoding : Decoding refers to the process of arriving at the equivalent English word from the code word given. Hence, we can look at two broad categories of questions in coding-decoding. In the 1st category of questions, a particular code is given and on the basis of this given code, we have to find out how another word (in English Language) can be coded. The correct code for the given word has to be selected from the answer choices on the basis of the code given in the question. Ex : In certain code if the word "VIRTUAL" is coded as "UHQSTZK", then in this code, how is the word "PAINFUL" coded? Solution :

By observation, we can find that in coding "VIRTUAL" as "UHQSTZK", each letter in the given word has been replaced by the letter that comes immediately before it in the English alphabet. Using this code, if we now have to code "PAINFUL", we need to replace each letter of the word with the letter that comes before it in the English alphabet. Thus the code will be "OZHMETK". Note : Here, it can be seen form the example, we treat the English alphabet in a circular fashion. i.e., the letter that comes after Z is A. Similarly, the letter that comes before A is Z. In the second category of questions, a particular code is given and on the basis of this given code, we have to find out the equivalent word in English language for a word given coded form. The correct word (in English Language) for the given coded word has to be selected from the answer choices on the basis of the code given in the question. Ex : In a particular Code, if the word :SYSTEM" is coded as "UAUVGO", then in teh same code, what does "HCUJKQP" stand for? Sol : Here, we can observe that each letter in the given word is replaced by the second letter that comes after it in the English alphabet to give us the word in coded form. Hence, to know what "HCUJKOP", we should replace each letter by a letter which second letter before it in the English alphabet. This gives us the word "FASHION". The first step to be taken in solving questions in coding - decoding is to crack the code. To do this, it will be helpful to understand the broad types of codes that are used. Ofcourse, the number of codes that can be created and used are infinite but one can always keep the commonly used codes in mind. Secondly, practice will make a student much more comfirtable and conversant with these types of questions. Use the letter that comes one or any fixed number of places before it in the alphabet. Use a letter as many places from the end of the alphabet as the original letter is from the begining of the alphabet. Each vowel may be replaced by the next vowel that comes in the alphabet and each consonant by another consonant following certain pattern with reference to the consonant under consideration. Use one code for all the letters in even places in the given word and a different code for letters n the odd places in the given word to give a new word. The same letters in the given word may be used in a cyclic or in some other order to give the coded word. Some numerical values can be attributed to the letters of the word based on the letter positions in the alphabet. Combination of two or more of the above ways of coding. Coding and Decoding Questions and Answers F sponsored links riends, in our last post we have discussed basic shortcut techniques of Coding - Decoding section of Reasoning (read that post here). Today we shall discuss some example problems of Coding and Decoding with detailed explanations. 1. If the word "DIAGRAM" is coded as "AGDMIRA", then the word "PICTURE" can be coded as ? 1. rpteiuc 2. ctpeiur 3. rtpeuic 4. ctpriuc 5. rpteicu Approach : The letters of the given word are written in a jumbled order to give us the word in the coded form. We need to find out the manner in which they have been jumbled. If you keenly observe the given words, the first letter of the word D has been used as the 3rd letter in the code. So, for the word PICTURE, the first letter P will have to be the third letter in the code. This eliminates choices (1) and (5), Then, the second letter I is used as the third from the last and that is not happening in choice (3). So, we now have choice (2) and (4). Since there are two As in the given word, let us leave aside A for the time being and look at some of the other letters. If we take the last letter M, it comes as the fourth letter in the code. SO, E in the word PICTURE should come as the fourth letter in the code. This

leaves us only choice (2). So the correct answer is option (2). Note : Here, suppose even this has not given us a unique choice, we would then look at the remaining letters too. 2. If the word "CODING" is represented as DPEJOH, then the word "CURFEW" can be represented as 1. dvsgfx 2. dvshfx 3. dgshfx 4. dtsgfy 5. dysgff Approach : Each letter in the given word "CODING" is replaced by the letter that immediately follows it in the English alphabet. Hence, the correct choice is option (1) 3. if "ASHTRAY" is coded as "DVKWUDB", then what does the word "UDQFLG" in that code mean? 1. RADISH 2. MANAGER 3. RUNNER 4. RANDOM 5. RANCID Approach : If we replace each letter of the word "ASHTRAY" by a letter which is three letters to the right of it in the alphabet, then we get "DVKWUDB". So, to find out the word whose code is "UDQFLG", we replace each letter in the code by a letter which comes three places behind in the alphabet. We then get "RANCID" as the word. Option (5). 4. If "RAJESH" is coded as "SZKDTG", then "PRANESH" should be coded as 1. QQBMDTG 2. QSBMDTG 3. QQBMFRI 4. QSZMDTG 5. QQZMFRI Approach :The first letter is shifted by one alphabet forward, the second by one alphabet backward, the third by one forward and so on... So, the word "PRANESH" will be coded as "QQBMFRI" which is option (3). 5. If "PROMPT" is coded as QSPLOS, then "PLAYER" should be coded as 1. qmbzfs 2. qmbxfq 3. okzxfq 4. qmbfqx 5. omzxfq Approach : The first half of the word has the letters being moved back by one letter in the alphabet and the second half of the word has the letters being moved forward by one letter in the alphabet. So, option (2). 6. If in a certain code, the number 1 is assigned to all letters in the even places in the alphabet and the number 2 is assigned to all letters in the odd places in the alphabet, then the code for the word ALPHABET will be 1. 21112121 2. 21121221 3. 21111221 4. 21112211 5. 21212121 Approach : An easy way to do these sort of problems is to Just remember the place values of the alphabets so that it will be easier for you to solve these type of problems. Read shortcut techniques to remember place values of alphabets from here. According to above link, ALPHABET can be written as A -- > 1 (Odd Number) so we have to assign value 2 L --> 12 (Even Number) so we have to assign value 1 P --> 16 (Even Number) so we have to assign value 1 H ---> 8 (Even Number) so we have to assign value 1 A -- > 1 (Odd Number) so we have to assign value 2 B--> 2 (Even Number) so we have to assign value 1 E--> 5 (Odd Number) so we have to assign value 2 T--> 20 (Even Number) so we have to assign value 1 So the answer is 21112121. Which is option (1). 7. In a certain code, a is represented by 1, b is by 2, c by 3 and so on; then all multiples of 2 are assigned a code of 2 and non-multiples of 2 are assigned a code of 1. In this scheme of coding, what would the word PAPERS be coded as ?

1. 121212 2. 212121 3. 212122 4. 121211 5. 212112 Approach : According to place values, the given word PAPERS can be written as P --> 16 (multiple of 2) so we can write 2 A---> 1 (non multiple of 2) so we can write 1 P ---> 16 (multiple of 2) so we can write 2 E ---> 5 (non multiple of 2) so we can write 1 R---> 18 (multiple of 2) so we can write 2. So the code will be 21212. Which is option (5). 8. If in a certain code "RANGE" is coded as 12345 and "RANDOM" is coded as 123678, then the code for the word "MANGO" would be 1. 82357 2. 84563 3. 82346 4. 82347 5. 82543 Approach : In these type of questions, we cannot get the code for all the letters of the alphabet but we can get the code for the letters of the word "MANGO" which is of interest to us. Take one by one of these letters of the word "MANGO" and look for the code of that specific letter from the words for which the code is already given to us. Here, in this case, when we observe that R, A and N are the first three letters of both the given words in the same order and the code for both words have 1, 2 and 3 in that order at the begining. Because of lack of any additional information, we then conclude that the code for R is 1, A is 2 and N is 3 and that the letters givne in the code are in the same order as the letters given in the word. Thus, we can make out that the code for MANGO is 82347 on the basis of the code different letters of the word can be given. Choice (3). That's all for now friends. In our next post we shall discuss some more hard problems on Coding and Decoding. All the best and Happy Reading :)

Anda mungkin juga menyukai